พิสูจน์ $\lim_{n\rightarrow\infty}\frac{n\log(n)}{\log(n!)} = 1$ [ซ้ำ]

Jan 25 2021

ฉันต้องการที่จะพิสูจน์ $$\lim_{n\rightarrow\infty}\frac{n\log(n)}{\log(n!)} = 1$$แต่ฉันสิ้นปัญญาแล้ว ฉันได้สำรวจเว็บแล้ว แต่ฉันสามารถหาตัวอย่าง / คำตอบที่พิสูจน์ว่าผกผัน = 0 และฉันได้ลองใช้วิธีการมากมายด้วยตัวเองโดยไม่เกิดประโยชน์ (คำขยาย, L'Hopitals Rule พร้อมรากศัพท์ที่แตกต่างกันนับไม่ถ้วนซึ่งผิดทั้งหมด) ใครช่วยชี้ทิศทางที่ถูกต้องได้ไหมผมติดหมดแล้ว ...

คำตอบ

3 BenjaminWang Jan 25 2021 at 08:52

โปรดทราบว่า $\log n! = \sum_{k=1}^n \log k$. โดยการวาดกราฟที่เกี่ยวข้องคุณสามารถดู:

$$\int_1^n \log x dx \le \sum_{k=1}^n \log k $$

$$\le \int_1^{n+1} \log x dx$$

ตอนนี้คำนวณอินทิกรัล $\int_1^m \log x dx = m \log m - m + 1$ดังนั้นข้างต้นจึงกลายเป็น

$$n \log n - n + 1 \le \log n! \le (n+1)\log(n+1)-n$$

ตอนนี้เราได้ผลลัพธ์ของคุณโดยการบีบทฤษฎีบทหลังจากหารผ่าน

1 crystal_math Jan 25 2021 at 09:04

$\log(n!)\ge \frac{n}{2}\log(\frac{n}{2})$ และอื่น ๆ $\dfrac{n\log(n)}{\log(n!)}\le \dfrac{n\log(n)}{\frac{n}{2}\log(\frac{n}{2})}$

เมื่อคุณประเมินขีด จำกัด ของขอบเขตบนนี้คุณจะได้รับ $2$ ตั้งแต่ $\lim_{n\rightarrow \infty} \dfrac{\log(n)}{\log(n/2)} = 1$. อย่างไรก็ตามหากคุณเลือก$\epsilon >1$คุณเห็น

$\log(n!)\ge \frac{n}{\epsilon}\log(\frac{n}{\epsilon})$ และอื่น ๆ $$\dfrac{n\log(n)}{\log(n!)}\le \dfrac{n\log(n)}{\frac{n}{\epsilon}\log(\frac{n}{\epsilon})}\rightarrow \epsilon$$

และตั้งแต่นั้นมา $\epsilon>1$ (โดยพลการ) คุณสามารถสรุปได้ $$\dfrac{n\log(n)}{\log(n!)}\le 1$$

(คุณสามารถรับขอบเขตล่างได้อย่างง่ายดาย) ดังนั้นขีด จำกัด จะต้องเป็น $1$.

1 zkutch Jan 25 2021 at 08:58

การใช้ $$\left( \frac{n}{e}\right)^n \lt n! \lt e \left( \frac{n}{2}\right)^n$$ เรามี $$n \log \frac{n}{e} \lt \log n! \lt \log e+ n \log \frac{n}{2}$$

ส่วนที่เพิ่มเข้าไป.

สำหรับด้านซ้ายขั้นตอนแรกของการเหนี่ยวนำนั้นชัดเจน แล้ว$$(n+1)!=n!(n+1) \gt \left( \frac{n}{e}\right)^n (n+1) = \\ =\left( \frac{n+1}{e}\right)^{n+1} \frac{(n+1)\left( \frac{n}{e}\right)^n}{\left( \frac{n+1}{e}\right)^{n+1}} \gt \left( \frac{n+1}{e}\right)^{n+1}$$ เพราะ $(n+1)\left( \frac{n}{e}\right)^n \left( \frac{n+1}{e}\right)^{-n-1}\gt 1$ เทียบเท่า $\left(1+ \frac{1}{n}\right)^{n} \lt e$.

สำหรับด้านขวา $$n! \lt \left(\frac{n+1}{2}\right)^{n} = e\left(\frac{n}{2}\right)^{n} \frac{\left(\frac{n+1}{2}\right)^{n}}{e\left(\frac{n}{2}\right)^{n}} = \\ =e\left(\frac{n}{2}\right)^{n} \frac{\left(1+ \frac{1}{n}\right)^{n}}{e} \lt e\left(\frac{n}{2}\right)^{n}$$

UNOwen Jan 25 2021 at 09:00

$$\displaystyle \frac{x\ln \left(x\right)}{\ln \left(x!\right)}=\frac{x\ln\left(x\right)}{\ln\left(\Gamma \left(x+1\right)\right)}$$

ใช้กฎของL'Hôpital

$$\lim _{x\to \infty }\left(\frac{x\ln \left(x\right)}{\ln \left(\Gamma \:\left(x+1\right)\right)}\right)=\lim_{x\to \:\infty \:}\left(\displaystyle \frac{\ln(x)+1}{\psi \:^{\left(0\right)}\left(x+1\right)}\right)$$

สมัครอีกครั้งให้ผลตอบแทน

$$\lim _{x\to \infty }\left(\frac{\frac{1}{x}}{\psi ^{\left(1\right)}\left(x+1\right)}\right)=\lim _{x\to \infty }\left(\frac{1}{x\left(\psi ^{\left(1\right)}\left(x+1\right)\right)}\right)$$

ตัวส่วนเข้าใกล้ 1 as $x\rightarrow \infty$.